1answer.
Ask question
Login Signup
Ask question
All categories
  • English
  • Mathematics
  • Social Studies
  • Business
  • History
  • Health
  • Geography
  • Biology
  • Physics
  • Chemistry
  • Computers and Technology
  • Arts
  • World Languages
  • Spanish
  • French
  • German
  • Advanced Placement (AP)
  • SAT
  • Medicine
  • Law
  • Engineering
Ksju [112]
3 years ago
9

Solve the system of equations algebraically.

Mathematics
2 answers:
Marta_Voda [28]3 years ago
7 0

Answer:

Choice d is correct answer.

Step-by-step explanation:

We have given two equation.

3x-3y = 6                eq(1)

6x-7y = 8                eq(2)

We have to find the solution of given equations.

Multiplying by both sides of eq(1) by 2, we have

2(3x-3y) = 2(6)

6x-6y = 12                  eq(3)

Subtracting eq(3) to eq(2) , we have

6x-6y -(6x-7y) = 12-8

6x-6y-6x+7y = 4

Adding like terms, we have

6x-6x+7y-6y = 4

y = 4

Putting the value of y in eq(1), we have

3x-3(4) = 6

3x-12 = 6

Adding 12 to both sides of above equation, we have

3x-12+12= 6+12

3x = 18

Dividing by 3 to both sides of above equation , we have

3x/3 = 18/3

x = 6

Hence, the solution set is (6,4).

algol133 years ago
5 0

Answer:

The correct option is d.

Step-by-step explanation:

The given equations are

3x-3y=6               ....(1)

6x-7y=8                ..... (2)

Multiply equation (1) by  by 2, to make same coefficient of x.

6x-6y=12               ....(3)

Subtract equation (2) form equation (3).

6x-6y-6x+7y=12-8

y=4

Substitute this value in equation (1),

3x-3(4)=6

3x-12=6

3x=18

x=6

The solution of the given system of equation is (6,4).

Therefore the correct option is d.

You might be interested in
Determine the gcf and lcm between 45,60 and 80
Ket [755]

Answer: LCM = 720, GCF = 5 Please consider Brainliest.

Step-by-step explanation:

Find the least multiple of all three numbers, it should be 720. Find the Greatest factor, which should work. But remember, 15 doesn't work because 80 doesn't have 15 as a factor.

3 0
2 years ago
Find the area of a circle that has a diameter of 60 feet.
IrinaK [193]

Answer:

60π

Step-by-step explanation:

Hullo :D

I think this will be a piece of helpful info:

<em>The formula for finding the area of a circle if πd.</em>

<em />

In that case, we just substitute 60 (our d)

So the area will be 60π

If you want that multiplied, you'll get 188.4 (multiplying by 3.14)

Thus, the answer is $\boxed{60\pi}.

3 0
3 years ago
EOC 2.27
Dovator [93]

The variable that cannot be illustrated by a stem-and-leaf plot is C. Number of doctor visits during the past year.

<h3>What is a stem and leaf plot?</h3>

It can be inferred that a stem and leaf plot simply means the technique that is used to classify continuous or discrete variables.

In this case, the variable that cannot be illustrated by a stem-and-leaf plot is the number of doctor visits during the past year. This is because it's an ordinal variable.

Learn more about stem and leaf plot on:

brainly.com/question/12276901

4 0
3 years ago
Which of the following equals 5.96
True [87]

Uhh could you provide some answer choices?

4 0
3 years ago
Please explain step by step
soldi70 [24.7K]

Answer:

23

Step-by-step explanation:

51+(2x+3)=100 ( exterior angle is equal to two opposite interior angles)

51+2x+3=100

2x+54=100

2x=100-54

x=46/2

x=23

7 0
3 years ago
Other questions:
  • A sale of 1/4 off the original price of $100
    11·2 answers
  • Please help me solve both these math problems #'s 4 and 5 just put the letter, Also the question is in the attachtment!
    9·1 answer
  • PLEASE HELP ME ASAP!
    11·2 answers
  • The graph shows the distance of a car from a measuring position located on the edge of a straight road.(a) What was the average
    15·1 answer
  • 8=72<br> 2=6<br> 5=30<br> 9=?<br><br> what is the value of ?
    13·2 answers
  • In a certain Algebra 2 class of 28 students, 23 of them play basketball and 12
    11·1 answer
  • What is 16 divided by 845?
    7·2 answers
  • PLEASE HELP ME QUICKLY
    8·2 answers
  • Please answer because I need this now!
    15·1 answer
  • If Fahmida randomly picks a jelly bean out of the jar, what is the probability that the jelly bean will be green (find the simpl
    11·1 answer
Add answer
Login
Not registered? Fast signup
Signup
Login Signup
Ask question!